Hepatic Dysfunction NCLEX Style Questions

Ace your homework & exams now with Quizwiz!

A patient with a history of postnecrotic cirrhosis is being cared for on your unit. What is this type of cirrhosis often the result of? A) A previous bout of acute viral hepatitis B) Chronic alcoholism C) Bilary obstruction D) Cholangitis

ANS: A Postnecrotic cirrhosis involves broad bands of scar tissue and is the result of a previous bout of acute viral hepatitis. Alcoholic cirrhosis is most frequently due to chronic alcoholism. Biliary cirrhosis usually results from chronic biliary obstruction and infection (cholangitis).

You are caring for a patient with a blocked bile duct from a tumor. What would you document about the patient's urine? A) Urine is dark amber and concentrated. B) Urine is orange and foamy. C) Urine is rust colored and concentrated. D) Urine is pale yellow and dilute.

ANS: B If the bile duct is obstructed, the bile will be reabsorbed into the blood and carried throughout the entire body. It is excreted in the urine, which becomes deep orange and foamy. Therefore options A, C, and D are incorrect.

What liver function test is a sensitive indicator of injury to liver cells and useful in detecting acute liver disease such as hepatitis? A) Clotting factors B) Serum aminotransferases C) GGT D) Alkaline phosphatase

ANS: B Serum aminotransferases (previously called transaminases) are sensitive indicators of injury to the liver cells and are useful in detecting acute liver disease such as hepatitis.

A patient is being discharged home after liver transplantation. The patient still has a hepatic artery catheter in place. What is this hepatic artery catheter for? A) To monitor portal hypertension B) To give immunosuppressive drugs during the first weeks after transplantation C) To monitor vascular changes in the hepatic system D) To deliver a continuous chemotherapeutic dose until completed

ANS: D In most cases, the hepatic artery catheter has been inserted surgically and has a prefilled infusion pump that delivers a continuous chemotherapeutic dose until completed. The hepatic artery catheter does not monitor portal hypertension, deliver immunosuppressive drugs, or monitor vascular changes in the hepatic system.

A nurse is performing an admission assessment for an 81-year-old patient who generally enjoys good health. When considering normal, age-related changes to hepatic function, the nurse should anticipate what finding? A) Similar liver size and texture as in younger adults B) A nonpalpable liver C) A slightly enlarged liver with palpably hard edges D) A slightly decreased size of the liver

Ans: D Feedback: The most common age-related change in the liver is a decrease in size and weight. The liver is usually still palpable, however, and is not expected to have hardened edges.

Which priority teaching information should the nurse discuss with the client to help prevent contracting hep. B? A. Explain the importance of good hand washing. B. Tell the client to take the hepatitis B vaccine in three (3) doses. C. Tell the client not to ingest unsanitary food or water. D. Discuss how to implement standard precautions.

Answer B Tell the client to take the hepatitis B vaccine in three (3) doses.

A client is admitted to the hospital with viral hepatitis, complaining of "no appetite" and "losing my taste for food." What instruction should the nurse give the client to provide adequate nutrition? " a. Select foods high in fat b. Increase intake of fluids, including juices. c. Eat a good supper when anorexia is not as severe. d. Eat less often, preferably only three large meals daily."

Answer B : Although no special diet is required to treat viral hepatitis, it is generally recommended that clients consume a low-fat diet because fat may be tolerated poorly because of decreased bile production. Small frequent meals are preferable and may even prevent nausea. Frequently, appetite is better in the morining, so it is easier to eat a good breakfast. An adequated fluid intake of 2500 to 3000 mL/day that includes nutritional juices is also important.

The client is in the preicteric phase of hepatitis. Which signs/symptoms would thenurse expect the client to exhibit during this phase? 1.Clay-colored stools and jaundice .2.Normal appetite and pruritus. 3.Being afebrile and left upper quadrant pain. 4.Complaints of fatigue and diarrhea.

Correct Answer 4 "Flu-like" symptoms are the first com-plaints of the client in the preicteric phase of hepatitis, which is the initial phase and may begin abruptly or insidiously

A patient has undergone a liver biopsy. Which of the following postprocedure positions is appropriate? a) On the right side b) Trendelenburg c) High Fowler's d) On the left side

On the right side Explanation: In this position, the liver capsule at the site of penetration is compressed against the chest wall, and the escape of blood or bile through the perforation made for the biopsy is impeded. Positioning the patient on his left side is not indicated. Positioning the patient in the Trendelenburg position may be indicated if the patient is in shock, but is not the position designed for the patient after liver biopsy. High Fowler's position is not indicated for the patient after liver biopsy.

Which of the following liver function studies is used to show the size of abdominal organs and the presence of masses? a) Angiography b) Ultrasonography c) Electroencephalogram d) Magnetic resonance imaging

Ultrasonography Explanation: A ultrasonography will show the size of the abdominal organs and the presence of masses. Magnetic resonance imaging is used to detect hepatic neoplasms. An angiography is used to visualize hepatic circulation and detect the presence and nature of hepatic masses. An electroencephalogram is used to detect abnormalities that occur with hepatic coma.

A client is admitted for suspected GI disease. Assessment data reveal muscle wasting, a decrease in chest and axillary hair, and increased bleeding tendency. The nurse suspects the client has: a) appendicitis. b) peptic ulcer disease. c) cholelithiasis. d) cirrhosis.

cirrhosis. Explanation: Muscle wasting, a decrease in chest and axillary hair, and increased bleeding tendencies are all symptoms of cirrhosis. The client may also have mild fever, edema, abdominal pain, and an enlarged liver. Clients with peptic ulcer disease complain of a dull, gnawing epigastric pain that's relieved by eating. Appendicitis is characterized by a periumbilical pain that moves to the right lower quadrant and rebound tenderness. Cholelithiasis is characterized by severe abdominal pain that presents several hours after a large meal.

"A patient with hepatitis B is being discharged in 2 days. In the discharge teaching plan the nurse should include instructions to "a. avoid alcohol for the first 3 weeks. B. use a condom during sexual intercourse. c. have family members get an injection of immunoglobulin. d. follow a low-protein, moderate-carbohydrate, moderate-fat diet."

"3. Correct answer: b Rationale: Hepatitis B virus may be transmitted by mucosal exposure to infectious blood, blood products, or other body fluids (e.g., semen, vaginal secretions, saliva). Hepatitis B is a sexually transmitted disease that is acquired through unprotected sex with an infected person. Condom use should be taught to patients to prevent transmission of hepatitis B."

A patient with hepatitis A is in the acute phase. The nurse plans care for the pateint based on the knowledge that: "a. pruritus is a common problem with jaundice in this phase. b. the pateint is most likely to transmit the disease during this phase. c. gastrointestinal symptoms are not severe in hepatitis A they are in hepatitis B. d. extrahepatic manifestations of glomerulonephritis and polyarteritis are common in this phase."

"Correct answer: a Rationale: The acute phase of jaundice may be icteric (i.e., symptomatic, including jaundice) or anicteric. Jaundice results when bilirubin diffuses into the tissues. Pruritus sometimes accompanies jaundice. Pruritus is the result of an accumulation of bile salts beneath the skin."

A 55-year-old female patient with hepatocellular carcinoma (HCC) is undergoing radiofrequency ablation. The nurse should recognize what goal of this treatment? A) Destruction of the patient's liver tumor B) Restoration of portal vein patency C) Destruction of a liver abscess D) Reversal of metastasis

Ans: A Feedback: Using radiofrequency ablation, a tumor up to 5 cm in size can be destroyed in one treatment session. This technique does not address circulatory function or abscess formation. It does not allow for the reversal of metastasis.

"A client has developed hepatitis A after eating contaminated oysters. The nurse assesses the client for which of the following? "1. Malaise 2. Dark stools 3. Weight gain 4. Left upper quadrant discomfort"

Correct Answer 1: Rationale: Hepatitis causes GI symptoms such as anorexia, nausea, right upper quadrant discomfort and weight loss. Fatigue and malaise are common. Stools will be light or clay colored if conjugated bilirubin is unable to flow out of the liver because of inflammation or obstruction of the bile ducts.

A client is hospitalized with hepatitis A. Which of the client's regular medications is contraindicated due to the current illness? 1. Prilosec (omeprazole) 2. Synthroid (levothyroxine) 3. Premarin (conjugated estrogens) 4. Lipitor (atorvastatin)

Correct: 4 Lipid-lowering agents are contraindicated in the client with active liver disease. Answers A, B, and C are incorrect because they are not contraindicated in the client with active liver disease.

The nurse is caring for a patient with cancer of the liver. The patient has a percutaneous biliary drainage system. What assessments should the nurse record in the patient's chart? A) Amount and color of drainage B) Temperature of drainage C) Odor of drainage D) Consistency of drainage

ANS: A The percutaneous biliary system is open to external drainage. The bile is observed closely for the amount, color, and presence of blood and debris. The nurse would not record the temperature, odor, and consistency of the drainage.

A nurse is caring for a client with cirrhosis. The nurse assesses the client at noon and discovers that the client is difficult to arouse and has an elevated serum ammonia level. The nurse should suspect which situation? a) The client didn't take his morning dose of lactulose (Cephulac). b) The client is relaxed and not in pain. c) The client's hepatic function is decreasing. d) The client is avoiding the nurse.

C) The client's hepatic function is decreasing The decreased level of consciousness caused by an increased serum ammonia level indicates hepatic disfunction. If the client didn't take his morning dose of lactulose, he wouldn't have elevated ammonia levels and decreased level of consciousness this soon. These assessment findings don't indicate that the client is relaxed or avoiding the nurse.

A client is suspected of having hepatitis. Which diagnoistic test result will assist in confirming this diagonis ? A.Elevate hemoglobin level B. Elevated serum bilirubin level C. Elevated blood urea nitrogen level D. Decreasd erythrocycte sedimentation rate

Correct Answer B Laboratory indicator of hepatitis include elevated liver enzyme levels, elevated serum bilirubin levels.Thinking about the organ that is involved in hepatitis should assist in directing to choose option B liver function test.

The home care nurse is visiting a client during an icteric phase of hepatitis of unknown etiology. The nurse would be MOST concerned if the client made which of the following comments? "1. ""I must not share eating utensils with my family."" 2. ""I must use my own bath towel."" 3. ""I'm glad that my husband and I can continue to have intimate relations."" 4. ""I must eat small, frequent feedings."""

Correct: 3 3. ""I'm glad my husband..."" - CORRECT: avoid sexual contact until serologic indicators return to normal

"A female client who has just been diagnosed with hepatitis A asks, "How could I have gotten this disease?" What is the nurse's best response? a. "You may have eaten contaminated restaurant food." b. "You could have gotten it by using I.V. drugs." c. "You must have received an infected blood transfusion." d. "You probably got it by engaging in unprotected sex.""

"Answer A. Hepatitis A virus typically is transmitted by the oral-fecal route — commonly by consuming food contaminated by infected food handlers. The virus isn't transmitted by the I.V. route, blood transfusions, or unprotected sex. Hepatitis B can be transmitted by I.V. drug use or blood transfusion. Hepatitis C can be transmitted by unprotected sex."

A physician orders spironolactone (Aldactone), 50 mg by mouth four times daily, for a client with fluid retention caused by cirrhosis. Which finding indicates that the drug is producing a therapeutic effect? a) Loss of 2.2 lb (1 kg) in 24 hours b) Serum potassium level of 3.5 mEq/L c) Blood pH of 7.25 d) Serum sodium level of 135 mEq/L

A) Loss of 2.2 lb (1 kg) in 24 hours Daily weight measurement is the most accurate indicator of fluid status; a loss of 2.2 lb (1 kg) indicates loss of 1 L of fluid. Because spironolactone is a diuretic, weight loss is the best indicator of its effectiveness. This client's serum potassium and sodium levels are normal. A blood pH of 7.25 indicates acidosis, an adverse reaction to spironolactone.

The triage nurse in the emergency department is assessing a patient who presented with complaints of not feeling well. The patient has ascites and an enlarged liver. The patient reports a history of drinking a 12 pack of beer every evening for the past 15 years. The nurse is aware that the patient is at risk for which disease? A) Cirrhosis B) Renal failure C) Hepatitis D) Cardiovascular disease

ANS: A Cirrhosis is a chronic disease characterized by replacement of normal liver tissue with diffuse fibrosis that disrupts the structure and function of the liver. There are three types of cirrhosis or scarring of the liver. This is most frequently due to chronic alcoholism. This patient's history does not put him or her at risk for renal failure, hepatitis, or cardiovascular disease.

A patient with a liver mass is undergoing a percutaneous liver biopsy. After the procedure the nurse assists the physician in positioning the patient. What position should they position the patient in? A) On the right side with a pillow under the costal margin B) Supine position C) On the left side with a pillow under the knees D) Trendelenberg position

ANS: A Immediately after a percutaneous liver biopsy, assist the patient to turn onto the right side and place a pillow under the costal margin. Instruct the patient to remain in this position, recumbent and immobile, for several hours. The patient will not be placed in the supine or the Trendelenberg position. Option C is incorrect.

A student asks the pathophysiology instructor what causes ascites. What would the instructor tell the student is involved in the physiologic process that causes ascites? (Mark all that apply.) A) Sodium and water retention B) A damaged liver C) Insufficient renal flow D) Increased lymphatic flow E) Decreased synthesis of immunoglobulin G

ANS: A, B, D Sodium and water retention, increased intravascular fluid volume, increased lymphatic flow, and decreased synthesis of albumin by the damaged liver all contribute to the movement of fluid from the vascular system into the peritoneal space. Insufficient renal blood flow and a decreased synthesis of immunoglobulin G do not play a part in the formation of ascites.

In end-stage liver disease, hypervolemia is always a problem. What nursing interventions would be most appropriate when caring for these patients? (Mark all that apply.) A) Administering diuretics B) Giving pain medications C) Implementing fluid restrictions D) Teaching nutrition E) Enhancing patient positioning

ANS: A, C, E Pulmonary compromise, which is always a potential complication of ESLD because of plasma volume excess, makes prevention of pulmonary complications an important role for the nurse. Administering diuretics, implementing fluid restrictions, and enhancing patient positioning can optimize pulmonary function.

A patient with bleeding esophageal varices is brought to the emergency department by the paramedics. An immediate endoscopy is performed. What nursing intervention is appropriate? A) Keep patient NPO until results of test are known. B) Keep patient NPO until gag reflex returns. C) Give anesthetic gargles until post-procedure soreness goes away. D) Give anesthetic lozenges 10 minutes before meals.

ANS: B After the examination, fluids are not given until the patient's gag reflex returns. Lozenges and gargles may be used to relieve throat discomfort if the patient's physical condition and mental status permit. Appropriate nursing interventions do not include option A, as the result of the test is known immediately; anesthetic gargles and lozenges are contraindicated until the gag reflex returns.

A patient with liver cancer is at the clinic to talk with the physician. The physician tells the patient that it is to risky to do surgery on their hepatic tumor. The patient asks the nurse why surgery is so risky for them. The nurse explains that while surgical resection of the liver tumor is possible in some patients, the risks associated with this procedure increase when the patient has a history of what? A) Oral contraceptive use B) Cirrhosis C) Hypertension D) Prior abdominal surgery

ANS: B Although surgical resection of the liver tumor is possible in some patients, the underlying cirrhosis is so prevalent in cancer of the liver that it increases the risks associated with surgery. Oral contraceptive use is associated with benign liver tumors not malignant tumors. Prior abdominal surgery may have resulted in abdominal scar tissue formation, but poses minor if any risk to the patient. Hypertension can be controlled by use of anti-hypertensive drugs and again, poses a minor risk to the patient.

The nurse is caring for a patient with metastasis of her cancer to the liver. The patient is undergoing chemotherapy by TACE. What does this chemotherapy procedure cause? A) Necrosis of tumor cells B) Embolization of tumor vessels C) Necrosis of tumor vessels D) Embolization of tumor cells

ANS: B Embolization of tumor vessels with chemotherapy (a process known as transarterial chemoembolization [TACE]) produces anoxic necrosis with high concentrations of trapped chemotherapeutic agents. This procedure does not work on the cellular level of the tumor.

A 55-year-old female patient is undergoing radiofrequency thermal ablation of her primary liver tumor. What does this treatment entail? A) A tube is inserted into the tumor and ablation is done by superheated water. B) A needle electrode is inserted into the tumor and heated by radiofrequency. C) A tube is inserted into the tumor and chemotherapy is placed into the tumor. D) A needle electrode is inserted into the tumor and killed by laser.

ANS: B In radiofrequency thermal ablation, a needle electrode is inserted into the liver tumor under imaging guidance. Radiofrequency energy passes through to the noninsulated needle tip, causing heat and tumor cell death from coagulation necrosis. No tube is inserted into the tumor.

A nurse practitioner is teaching a health class in the local high school. The nurse practitioner informs the class about hepatitis B. What occupation does the nurse practitioner inform the class is at the greatest risk for contracting hepatitis B? A) Flight attendants B) Health care workers C) Fire fighters D) Educators

ANS: B People who are at high risk, including nurses and other health care personnel exposed to blood or blood products, should receive active immunization. Health care workers who have had frequent contact with blood are screened for anti-HBs to determine whether immunity is already present from previous exposure. Flight attendants, fire fighters and educators are not at high risk for hepatitis B.

Toxic hepatitis has a poor prognosis if there is a prolonged period between the exposure to the toxin and the onset of symptoms. There are no effective antidotes. What is the clinical course of toxic hepatitis? 1. Fever rises. 2. Hematemisis. 3. Clotting abnormalities. 4. Vascular collapse. 5. Coma. A) 12543 B) 12345 C) 23145 D) 31254

ANS: B Recovery from acute toxic hepatitis is rapid if the hepatotoxin is identified early and removed or if exposure to the agent has been limited. Recovery is unlikely if there is a prolonged period between exposure and onset of symptoms. There are no effective antidotes. The fever rises; the patient becomes toxic and prostrated. Vomiting may be persistent, with the emesis containing blood. Clotting abnormalities may be severe, and hemorrhages may appear under the skin. The severe GI symptoms may lead to vascular collapse. Delirium, coma, and seizures develop, and within a few days the patient may die of fulminant hepatic failure (discussed later) unless he or she receives a liver transplant.

A patient has come to the clinic with vague complaints that made the physician suspect liver disease. Laboratory work shows elevated GGT levels. What can elevated GGT levels indicate? A) Hemolytic jaundice B) Cirrohsis C) Liver cell dysfunction D) Viral hepatitis

ANS: C Increased GGT levels are associated with cholestasis but can also be due to alcoholic liver disease. Although the kidney has the highest level of the enzyme, the liver is considered the source of normal serum activity. The test determines liver cell dysfunction and is a sensitive indicator of cholestasis. An elevated GGT would not indicate hemolytic jaundice, cirrohsis, or viral hepatitis.

A patient with portal hypertension has been admitted to the floor you work on. What will you assess for related to portal hypertension? A) Bowel obstruction B) Vitamin A deficiency C) Ascites D) Hepatic encephalopathy

ANS: C Obstruction to blood flow through the damaged liver results in increased blood pressure (portal hypertension) throughout the portal venous system. This can result in varices and ascites in the abdominal cavity. Portal hypertension does not generally cause bowel obstruction. Vitamin A deficiency is not something the nurse assesses for. Hepatic encephalopathy is a complication of cirrohsis.

The nurse is caring for a patient with hepatic encephalopathy. The nurse's assessment reveals that the patient exhibits episodes of confusion, is difficult to arouse from sleep, has rigid extremities, and shows EEG abnormalities. Based upon these clinical findings, what is the patient's stage of hepatic encephalopathy? A) Stage 1 B) Stage 2 C) Stage 3 D) Stage 4

ANS: C Patients in the third stage of hepatic encephalopathy exhibit the following symptoms: stuporous; difficult to arouse; sleeps most of the time; exhibits marked confusion; incoherent in speech; asterixis; increased deep tendon reflexes; rigidity of extremities; marked EEG abnormalities. Patients in stages 1 and 2 exhibit clinical symptoms that are not as advanced as found in stage 3, and patients in stage 4 are comatose. In stage 4, there is an absence of asterixis, absence of deep tendon reflexes, flaccidity of extremities, and EEG abnormalities.

The nursing instructor is teaching the senior nursing students about liver disease. What would the instructor teach the students is the most common surgical procedure for liver cancer? A) Cryosurgery B) Liver transplantation C) A lobectomy D) Laser hyperthermia

ANS: C Removal of a lobe of the liver (lobectomy) is the most common surgical procedure for excising a liver tumor. While liver transplantation and cryosurgery are other surgical options for management of liver cancer, these procedures are not performed at the same frequency as a lobectomy. Laser hyperthermia is a nonsurgical treatment for liver cancer.

A nurse is caring for a patient with severe hemolytic jaundice. Laboratory tests show free bilirubin to be 24 mg/dL. What is this patient at risk for? A) Chronic jaundice B) Pigment stones in portal circulation C) Brainstem damage D) Hepatomegaly

ANS: C Brainstem damage Prolonged jaundice, even if mild, predisposes to the formation of pigment stones in the gallbladder, and extremely severe jaundice (levels of free bilirubin exceeding 20 to 25 mg/dL) poses a risk for brainstem damage.

A local public health nurse is informed that a cook in a local restaurant has been diagnosed with hepatitis A. What should the nurse advise individuals who ate at this restaurant and have never received the hepatitis A vaccine? A) The hepatitis A vaccine B) The hepatitis B vaccine C) The hepatitis A and B vaccines D) An immune globulin injection

ANS: D For people who have not been previously vaccinated, hepatitis A can be prevented by the intramuscular administration of globulin during the incubation period, if given within 2 weeks of exposure. Administration of the hepatitis A vaccine will not protect the patient exposed to hepatitis A, as protection will take a few weeks to develop after the first dose of the vaccine. The hepatitis B vaccine provides protection again the hepatitis B virus, but plays no role in protection for the patient exposed to hepatitis A.

A nurse is caring for a patient with cancer of the liver whose condition has required the insertion of a percutaneous biliary drainage system. The nurse's most recent assessment reveals the presence of dark green fluid in the collection container. What is the nurse's best response to this assessment finding? A) Document the presence of normal bile output. B) Irrigate the drainage system with normal saline as ordered. C) Aspirate a sample of the drainage for culture. D) Promptly report this assessment finding to the primary care provider.

Ans: A Feedback: Bile is usually a dark green or brownish-yellow color, so this would constitute an expected assessment finding, with no other action necessary.

A patient's physician has ordered a "liver panel" in response to the patient's development of jaundice. When reviewing the results of this laboratory testing, the nurse should expect to review what blood tests? Select all that apply. A) Alanine aminotransferase (ALT) B) C-reactive protein (CRP) C) Gamma-glutamyl transferase (GGT) D) Aspartate aminotransferase (AST) E) B-type natriuretic peptide (BNP)

Ans: A, C, D Feedback: Liver function testing includes GGT, ALT, and AST. CRP addresses the presence of generalized inflammation and BNP is relevant to heart failure; neither is included in a liver panel.

"A female client who has just been diagnosed with hepatitis A asks, "How could I have gotten this disease?" What is the nurse's best response? "a. "You may have eaten contaminated restaurant food." b. "You could have gotten it by using I.V. drugs." c. "You must have received an infected blood transfusion." d. "You probably got it by engaging in unprotected sex.""

Answer A Hepatitis A virus typically is transmitted by the oral-fecal route — commonly by consuming food contaminated by infected food handlers. The virus isn't transmitted by the I.V. route, blood transfusions, or unprotected sex. Hepatitis B can be transmitted by I.V. drug use or blood transfusion. Hepatitis C can be transmitted by unprotected sex."

A patient with esophageal varices has medication ordered to decrease portal pressure and aid in preventing a first bleeding episode. What type of drug would the physician order? A) Antihypertensive agents B) Beta-blocking agents C) Calcium channel blockers D) Histamine-2 antagonists

ANS: B Propranolol (Inderal) and nadolol (Corgard), beta-blocking agents that decrease portal pressure, are the most common medications used both to prevent a first bleeding episode in patients with known varices and to prevent rebleeding. This makes options A, C, and D incorrect.

A critical care nurse is caring for a patient post liver transplantation. The patient has a fall in blood glucose level. What would the nurse do in this instance? A) Give an ampule of D50 every 4 hours B) Monitor an infusion of 5% glucose for the first 24 hours C) Monitor an infusion of 10% glucose for the first 48 hours D) Give an ampule of D50 every 2 hours

ANS: C Metabolic abnormalities require careful attention. A constant infusion of 10% glucose may be required in the first 48 hours to prevent a precipitous fall in the blood glucose level that results from decreased gluconeogenesis. Options A, B, and D are incorrect.

The nurse is caring for a patient with liver failure. The nurse understands that patients in liver failure often require vitamin therapy. Which vitamin does the liver require for the synthesis of prothrombin? A) Vitamin B12 B) Vitamin A C) Vitamin D D) Vitamin K

ANS: D The liver requires vitamin K for the synthesis of prothrombin and some of the other clotting factors. Breakdown of fatty acids into ketone bodies occurs primarily when the availability of metabolism is limited, as during starvation or in uncontrolled diabetes. Vitamins stored in the liver include A, B12, D, and several of the B-complex vitamins.

A nurse is caring for a patient who has been admitted for the treatment of advanced cirrhosis. What assessment should the nurse prioritize in this patient's plan of care? A) Measurement of abdominal girth and body weight B) Assessment for variceal bleeding C) Assessment for signs and symptoms of jaundice D) Monitoring of results of liver function testing

Ans: B Feedback: Esophageal varices are a major cause of mortality in patients with uncompensated cirrhosis. Consequently, this should be a focus of the nurse's assessments and should be prioritized over the other listed assessments, even though each should be performed.

"A patient with hepatitis B is being discharged in 2 days. In the discharge teaching plan the nurse should include instructions to: "A. Avoid alcohol for the first 3 weeks B.use condoms during sexual intercourse C. have family members get an injection of immunoglobulin D. follow low protein, moderate carb, moderate fat diet"

Correct: B B. is the correct answer as it is important to instruct the patient they this disease can be spread through sexual contact

A physician has ordered a liver biopsy for a client whose condition is deteriorating. Which of the following places the client at high risk due to her altered liver function during the biopsy? a) Low platelet count b) Low hemoglobin c) Decreased prothrombin time d) Low sodium level

A) Low platelet count Certain blood tests provide information about liver function. Prolonged prothrombin time (PT) and low platelet count place the client at high risk for hemorrhage. The client may receive intravenous (IV) administration of vitamin K or infusions of platelets before liver biopsy to reduce the risk of bleeding.

The nurse is caring for a client with a diagnosis of hepatitis who is experiencing pruritis. Which would be the most appropriate nursing intervention? A.Suggest that the client take warm showers. B.Add baby oil to the client's bath water. C.Apply powder to the client's skin. D.Suggest a hot-water rinse after bathing.

Answer B. Applying baby oil could help soothe the itchy skin. Answer A, C, and D would increase dryness and worsen the itching.

A nurse is assessing a 77-year-old male admitted to the unit for suspected liver dysfunction. What assessment finding will the nurse expect to find? A) Liver function results change dramatically in the older adult. B) Metabolism of medications is delayed primarily by the liver. C) The liver is enlarged with hard edges. D) The liver is decreased in size and weight.

ANS: D The most common change in the liver is a decrease in size and weight. Results of liver function test results do not normally change in the elderly. Metabolism of medication is decreased by intestinal absorption, renal excretion, and altered body distribution of fat, as well as a decrease in liver function.

A patient with liver cancer is undergoing radiation therapy. The patient asks the nurse if the radiation therapy is going to cure the cancer. What is the best answer the nurse could give? A) "The radiation therapy will hopefully prolong your life, but the major effect is palliative." B) "The radiation therapy is going to give you a second chance at life." C) "The radiation therapy will cure your cancer." D) "The radiation therapy will take away your pain and discomfort."

ANS: A Radiation therapy and chemotherapy have been used to treat cancer of the liver with varying degrees of success. Although these therapies may prolong survival and improve quality of life by reducing pain and discomfort, their major effect is palliative. The only complete answer is option

A patient who has undergone liver transplantation is ready to be discharged home. The nurse is providing discharge teaching. Which topic will the nurse emphasize the most related to discharge teaching? A) The patient will obtain measurement of drainage from the T-tube. B) The patient will exercise three times a week. C) The patient will take immunosuppressive agents as required. D) The patient will monitor for signs of liver dysfunction.

ANS: C The patient is given written and verbal instructions about immunosuppressive agent doses and dosing schedules. The patient is also instructed on steps to follow to assure that an adequate supply of medication is available so that there is no chance of running out of the medication or skipping a dose. Failure to take medications as instructed may precipitate rejection. The nurse would not teach the patient to measure drainage from a T-tube as they wouldn't go home with a T-tube. The nurse may teach the patient about the need to exercise or what the signs of liver dysfunction are, but the nurse would not stress them like they would stress the immunosuppressive drug regimen.

A patient is brought to the emergency department by ambulance. He has hematemesis and alteration in mental status. The patient has tachycardia, cool clammy skin, and hypotension. The patient has a history of alcohol abuse. What would the nurse suspect the patient has? A) Hemolytic jaundice B) Hepatic insufficiency C) Bleeding esophageal varices D) Portal hypertension

ANS: C The patient with bleeding esophageal varices may present with hematemesis, melena, or general deterioration in mental or physical status and often has a history of alcohol abuse. Signs and symptoms of shock (cool clammy skin, hypotension, tachycardia) may be present. The scenario does not describe hemolytic jaundice, hepatic insufficiency, or portal hypertension.

The nursing instructor is teaching the beginning nursing students how to palpate a liver. What technique will the student nurse use to palpate a patient's liver? A) Place hand under left lower rib cage and press down lightly with the other hand. B) Place the left hand over the abdomen and behind the left side at the 11th rib. C) Place hand under right lower rib cage and press down lightly with the other hand. D) Hold hand 90 degrees to abdomen and push down firmly.

ANS: C To palpate the liver, the examiner places one hand under the right lower rib cage and presses downward with light pressure with the other hand. Therefore options A, B, and D are incorrect.

The critical care nurse is caring for a patient with cirrhosis. What is an essential nursing function when caring for a patient with cirrhosis? A) Monitoring the patient's oral intake B) Monitoring the patient's support network C) Monitoring the patient for signs of hypervolemia D) Monitoring the patient's mental status

ANS: D Monitoring is an essential nursing function to identify early deterioration in mental status. The nurse monitors the patient's mental status closely and reports changes so that treatment of encephalopathy can be initiated promptly. An extensive neurologic evaluation is key to identify progression through the four stages of encephalopathy. The nurse would monitor the oral intake and watch for signs of hypervolemia, but they are not as essential as the patient's mental status because of the encephalopathy that goes with cirrhosis. Monitoring the support network is not essential at this time.

The nurse is caring for a patient with hepatic encephalopathy. While making the initial shift assessment the nurse notes that the patient has a flapping tremor of the hands. What will the nurse document this condition as in the patient's chart? A) Asterixis B) Constructional apraxia C) Fetor hepaticus D) Palmar erythema

ANS: A The nurse will document that a patient exhibiting a flapping tremor of the hands is demonstrating asterixis. While constructional apraxia is a motor disturbance, it is the inability to reproduce a simple figure. Fetor hepaticus is a sweet, slightly fecal odor to the breath and not associated with a motor disturbance. Skin changes associated with liver dysfunction may include palmar erythema, which is a reddening of the palms, but is not a flapping tremor.

A patient with esophageal varices is being cared for in the ICU. The varices have begun to bleed and the patient is at risk for hypovolemia. The patient has Ringer's lactate at 150 cc/hr infusing. What else might the nurse expect to have ordered to maintain volume for this patient? A) Arterial line B) Central venous catheter C) Foley catheter D) Volume expanders

ANS: D Because patients with bleeding esophageal varices have intravascular volume depletion and are subject to electrolyte imbalance, IV fluids with electrolytes and volume expanders are provided to restore fluid volume and replace electrolytes. Transfusion of blood components also may be required. An arterial catheter, a central venous catheter, and a Foley catheter will probably be ordered, but they won't maintain the patient's volume.

The nurse is reviewing the history of a patient newly admitted to the unit. This patient has been ordered vasopressin for bleeding esophageal varices. The nurse calls the physician to question the use of vasopressin when she reads that the patient has a history of what? A) Diabetes mellitus B) Chronic kidney disease C) Arthritis D) Coronary artery disease

ANS: D Coronary artery disease is a contraindication to the use of vasopressin, because coronary vasoconstriction is a side effect that may precipitate myocardial infarction. A history of diabetes, chronic kidney disease, or arthritis does not contraindicate the use of vasopressin.

A young woman is being discharged after a liver transplant. The young woman asks the nurse when she will be able to try to become pregnant. What would be an appropriate response by the nurse? A) "You will never have a period after a liver transplant and you will never be able to get pregnant." B) "You can consider trying to get pregnant now. Let's start you on prenatal vitamins today." C) "You should see a fertility specialist to discuss this topic." D) "Pregnancy can be considered 1 year after transplantation, but it is considered a high-risk pregnancy and you will need to discuss this with your doctor."

ANS: D The nurse should immediately address the patient's question by stating that research indicates that pregnancy can be considered 1 year after transplantation. The patient should also be informed that successful pregnancy outcomes have been reported, but these pregnancies carry a risk for the mother and infant and are considered high-risk pregnancies. The female patient should be informed that she should use a reliable form of birth control as she will have her menses and be able to get pregnant and pregnancy should be avoided for at least 1 year.

A client with cirrhosis is at risk for developing esophageal varices. Which of the following instructions should a nurse provide the client to minimize such risk? a) Increase intake of potassium-rich food. b) Avoid intake of sodium-rich food. c) Abstain from drinking alcohol. d) Use aspirin at least once a day.

Abstain from drinking alcohol. Explanation: A soft diet and elimination of alcohol, aspirin, and other locally irritating substances minimize the risk for developing esophageal varices. Intake of sodium- or potassium-rich food has no effect on the formation of varices.

A nurse is caring for a patient with hepatic encephalopathy. While making the initial shift assessment, the nurse notes that the patient has a flapping tremor of the hands. The nurse should document the presence of what sign of liver disease? A) Asterixis B) Constructional apraxia C) Fetor hepaticus D) Palmar erythema

Ans: A Feedback: The nurse will document that a patient exhibiting a flapping tremor of the hands is demonstrating asterixis. While constructional apraxia is a motor disturbance, it is the inability to reproduce a simple figure. Fetor hepaticus is a sweet, slightly fecal odor to the breath and not associated with a motor disturbance. Skin changes associated with liver dysfunction may include palmar erythema, which is a reddening of the palms, but is not a flapping tremor.

33. A patient with end-stage liver disease has developed hypervolemia. What nursing interventions would be most appropriate when addressing the patient's fluid volume excess? Select all that apply. A) Administering diuretics B) Administering calcium channel blockers C) Implementing fluid restrictions D) Implementing a 1500 kcal/day restriction E) Enhancing patient positioning

Ans: A, C, E Feedback: Administering diuretics, implementing fluid restrictions, and enhancing patient positioning can optimize the management of fluid volume excess. Calcium channel blockers and calorie restriction do not address this problem.

A patient with a diagnosis of esophageal varices has undergone endoscopy to gauge the progression of this complication of liver disease. Following the completion of this diagnostic test, what nursing intervention should the nurse perform? A) Keep patient NPO until the results of test are known. B) Keep patient NPO until the patient's gag reflex returns. C) Administer analgesia until post-procedure tenderness is relieved. D) Give the patient a cold beverage to promote swallowing ability.

Ans: B Feedback: After the examination, fluids are not given until the patient's gag reflex returns. Lozenges and gargles may be used to relieve throat discomfort if the patient's physical condition and mental status permit. The result of the test is known immediately. Food and fluids are contraindicated until the gag reflex returns.

A patient with a history of injection drug use has been diagnosed with hepatitis C. When collaborating with the care team to plan this patient's treatment, the nurse should anticipate what intervention? A) Administration of immune globulins B) A regimen of antiviral medications C) Rest and watchful waiting D) Administration of fresh-frozen plasma (FFP)

Ans: B Feedback: There is no benefit from rest, diet, or vitamin supplements in HCV treatment. Studies have demonstrated that a combination of two antiviral agents, Peg-interferon and ribavirin (Rebetol), is effective in producing improvement in patients with hepatitis C and in treating relapses. Immune globulins and FFP are not indicated.

A patient is suspected of having hepatitis. Which diagnostic test result will assist in confirming this diagnosis? a. Elevated hemoglobin level, b. Elevated serum bilirubin level, c. Elevated blood urea nitrogen level, d. Decreased erythrocyte sedimentation rate

Correct B Laboratory indicators of hepatitis include elevated liver enzyme levels, elevated serum bilirubin leveles, elevated erythrocyte sedimentatation rates, and leukopenia. An elevated blood urea nitrogen level may indicate renal dysfunction. A hemoglobin level is unrelated to this diagnosis.

The home care nurse is visiting a client with a diagnosis of hepatitis of unknown etiology. The nurse knows that teaching has been successful if the patient makes which on of the following statements? "1. ""I am so sad that I am not able to hold my baby."" 2."" I will eat after my family eats."" 3. ""I will make sure that my children don't eat or drink after me."" 4. ""I'm glad that I don't have to get help taking care of my children."""

Correct: 3 "1. not spread by casual contact 2. can eat together, but not share utensils 3. to prevent transmission - do not share eating utensils or drinking glasses, wash hands before eating and after using toilet 4. alternate rest/activity to promote hepatic healing, mother of young children will need help"

The health care provider orders lactulose for a patient with hepatic encephalopathy. The nurse will monitor for effectiveness of this medication for this patient by assessing which of the following? A) Relief of constipation B) Relief of abdominal pain C) Decreased liver enzymes D) Decreased ammonia levels

D) Decreased ammonia levels Hepatic encephalopathy is a complication of liver disease and is associated with elevated serum ammonia levels. Lactulose traps ammonia in the intestinal tract. Its laxative effect then expels the ammonia from the colon, resulting in decreased serum ammonia levels and correction of hepatic encephalopathy.

A patient with type 2 diabetes and cirrhosis asks the nurse if it would be okay to take silymarin (milk thistle) to help minimize liver damage. The nurse responds based on knowledge that A) Milk thistle may affect liver enzymes and thus alter drug metabolism. B) Milk thistle is generally safe in recommended doses for up to 10 years. C) There is unclear scientific evidence for the use of milk thistle in treating cirrhosis. D) Milk thistle may elevate the serum glucose levels and is thus contraindicated in diabetes.

A) Milk thistle may affect liver enzymes and thus alter drug metabolism There is good scientific evidence for the use of milk thistle as an antioxidant to protect the liver cells from toxic damage in the treatment of cirrhosis. It is noted to be safe for up to 6 years, not 10 years, and it may lower, not elevate, blood glucose levels. It does affect liver enzymes and thus could alter drug metabolism. Therefore patients will need to be monitored for drug interactions.

A client with liver failure has severe ascites. On initial shift rounds, his primary nurse finds his indwelling urinary catheter collection bag too full to store more urine. The nurse empties more than 2,000 ml from the collection bag. One hour later, she finds the collection bag full again. The nurse notifies the physician, who suspects that a bladder rupture is allowing the drainage of peritoneal fluid. The physician orders a urinalysis to be obtained immediately. The presence of which substance is considered abnormal? a) Albumin b) Chloride c) Urobilinogen d) Creatinine

A) Albumin Albumin is an abnormal finding in a routine urine specimen. Ascites present in liver failure contain albumin; therefore, if the bladder ruptured, ascites containing albumin would drain from the indwelling urinary catheter because the catheter is no longer contained in the bladder. Creatinine, urobilinogen, and chloride are normally found in urine.

Patients diagnosed with esophageal varices are at risk for hemorrhagic shock. Which of the following is a sign of potential hypovolemia? a) Hypotension b) Bradycardia c) Polyuria d) Warm moist skin

A) Hypotension Signs of potential hypovolemia include cool, clammy skin, tachycardia, decreased blood pressure, and decreased urine output.

When caring for a patient with a biliary obstruction, the nurse will anticipate administering which of the following vitamin supplements (select all that apply)? A) Vitamin A B) Vitamin D C) Vitamin E D) Vitamin K E) Vitamin B

A,B,C,D Biliary obstruction prevents bile from entering the small intestine and thus prevents the absorption of fat-soluble vitamins. Vitamins A, D, E, and K are all fat soluble and thus would need to be supplemented in a patient with biliary obstruction.

When caring for a patient with liver disease, the nurse recognizes the need to prevent bleeding resulting from altered clotting factors and rupture of varices. Which of the following nursing interventions would be appropriate to achieve this outcome (select all that apply)? A) Use smallest gauge possible when giving injections or drawing blood. B) Teach patient to avoid straining at stool, vigorous blowing of nose, and coughing. C) Advise patient to use soft-bristle toothbrush and avoid ingestion of irritating food. D) Apply gentle pressure for the shortest possible time period after performing venipuncture. E) Instruct patient to avoid aspirin and NSAIDs to prevent hemorrhage when varices are present.

A,B,C,E Using the smallest gauge for injections will minimize the risk of bleeding into the tissues. Avoiding straining, nose blowing, and coughing will reduce the risk of hemorrhage at these sites. The use of a soft-bristle toothbrush and avoidance of irritating food will reduce injury to highly vascular mucous membranes. The nurse should apply gentle but prolonged pressure to venipuncture sites to minimize the risk of bleeding.

A client with severe and chronic liver disease is showing manifestations related to inadequate vitamin intake and metabolism. He reports difficulty driving at night because he cannot see well. Which of the following vitamins is most likely deficient for this client? a) Vitamin K b) Vitamin A c) Riboflavin d) Thiamine

B) Vitamin A Problems common to clients with severe chronic liver dysfunction result from inadequate intake of sufficient vitamins. Vitamin A deficiency results in night blindness and eye and skin changes. Thiamine deficiency can lead to beriberi, polyneuritis, and Wernicke-Korsakoff psychosis. Riboflavin deficiency results in characteristic skin and mucous membrane lesions. Vitamin K deficiency can cause hypoprothrombinemia, characterized by spontaneous bleeding and ecchymoses.

The nurse is caring for a client with cirrhosis. Which assessment findings indicate that the client has deficient vitamin K absorption caused by this hepatic disease? a) Ascites and orthopnea b) Gynecomastia and testicular atrophy c) Purpura and petechiae d) Dyspnea and fatigue

C) Purpura and petechiae A hepatic disorder, such as cirrhosis, may disrupt the liver's normal use of vitamin K to produce prothrombin (a clotting factor). Consequently, the nurse should monitor the client for signs of bleeding, including purpura and petechiae. Dyspnea and fatigue suggest anemia. Ascites and orthopnea are unrelated to vitamin K absorption. Gynecomastia and testicular atrophy result from decreased estrogen metabolism by the diseased liver.

A client with hepatitis who has not responded to medical treatment is scheduled for a liver transplant. Which of the following most likely would be ordered? a) Chenodiol b) Ursodiol c) Tacrolimus d) Interferon alfa-2b, recombinant

C) Tacrolimus In preparation for a liver transplant, a client receives immunosuppressants to reduce the risk for organ rejection. Tacrolimus or cyclosporine are two immunosuppresants that may be used. Chenodiol and ursodiol are agents used to dissolve gall stones. Recombinant interferon alfa-2b is used to treat chronic hepatitis B, C, and D to force the virus into remission.

Which of the following indicates an overdose of lactulose? a) Hypoactive bowel sounds b) Constipation c) Watery diarrhea d) Fecal impaction

C) Watery diarrhea The patient receiving lactulose is monitored closely for the development of watery diarrheal stool, which indicates a medication overdose.

"When planning care for a patient with cirrhosis, the nurse will give highest priority to which of the following nursing diagnoses? A: Imbalanced nutrition: less than body requirements B: Impaired skin integrity related to edema, ascites, and pruritis C: Ecess fluid volume related to portal hypertension and hyperaldosteronism D: Ineffective breathing pattern related to pressure on diaphragm and reduced lung volume

CORRECT: D Although all of these nursing diagnoses are appropriate and important in the care of a patient with cirrhosis, AIRWAY and BREATHING are always the highest priorities.

A nurse is assisting with a percutaneous liver biopsy. Place the steps involved in care in the correct sequence from first to last. Ensure that the biopsy equipment is assembled and in order. Help the client assume a supine position. Make sure that the specimen container is labeled and delivered to the laboratory. While the physician inserts the needle, instruct the client to take a deep breath and hold it to keep the liver as near to the abdominal wall as possible. Place a rolled towel beneath the client's right lower ribs.

Ensure that the biopsy equipment is assembled and in order. Help the client assume a supine position. Place a rolled towel beneath the client's right lower ribs. While the physician inserts the needle, instruct the client to take a deep breath and hold it to keep the liver as near to the abdominal wall as possible. Make sure that the specimen container is labeled and delivered to the laboratory. When assisting with a percutaneous liver biopsy, the nurse ensures that the biopsy equipment is assembled and in order. He or she helps the client assume a supine position with a rolled towel beneath the right lower ribs. Before the physician inserts the needle, the nurse instructs the client to take a deep breath and hold it to keep the liver as near to the abdominal wall as possible. After specimen cells are obtained, they are placed in a preservative. The nurse makes sure that the specimen container is labeled and delivered to the laboratory.

Which type of deficiency results in macrocytic anemia? a) Folic acid b) Vitamin A c) Vitamin C d) Vitamin K

Folic acid Explanation: Folic acid deficiency results in macrocytic anemia. Vitamin C deficiency results in hemorrhagic lesions of scurvy. Vitamin A deficiency results in night blindness and eye and skin changes. Vitamin K deficiency results in hypoprothrombinemia, which is characterized by spontaneous bleeding and ecchymosis.

Which of the following would be the least important assessment in a patient diagnosed with ascites? a) Palpation of abdomen for a fluid shift b) Measurement of abdominal girth c) Foul-smelling breath d) Weight

Foul-smelling breath Explanation: Foul-smelling breath would not be considered an important assessment for this patient. Measurement of abdominal girth, weight, and palpation of the abdomen for a fluid shift are all important assessment parameters for the patient diagnosed with ascites.

Which type of jaundice seen in adults is the result of increased destruction of red blood cells? a) Hepatocellular b) Nonobstructive c) Obstructive d) Hemolytic

Hemolytic Explanation: Hemolytic jaundice results because, although the liver is functioning normally, it cannot excrete the bilirubin as quickly as it is formed. Obstructive jaundice is the result of liver disease. Nonobstructive jaundice occurs with hepatitis. Hepatocellular jaundice is the result of liver disease.

Patients diagnosed with esophageal varices are at risk for hemorrhagic shock. Which of the following is a sign of potential hypovolemia? a) Bradycardia b) Warm moist skin c) Hypotension d) Polyuria

Hypotension Explanation: Signs of potential hypovolemia include cool, clammy skin, tachycardia, decreased blood pressure, and decreased urine output.

A nurse is reviewing laboratory test results from a client. The report indicates that the client has jaundice. What serum bilirubin level must the client's finding exceed? Enter the correct number only.

Jaundice becomes clinically evident when the serum bilirubin level exceeds 2.5 mg/dL (43 fmol/L).

Which of the following indicates an overdose of lactulose? a) Constipation b) Watery diarrhea c) Hypoactive bowel sounds d) Fecal impaction

Watery diarrhea Explanation: The patient receiving lactulose is monitored closely for the development of watery diarrheal stool, which indicates a medication overdose.

The nurse is performing an assessment on a client bein evaluated for viral hepatitis. Which symptom will the nurse most likely assess on this client? 1. Arthralgia 2. Excitability 3. Headache 4. Polyphagia

"ANSWER: 1 Rationale: arthralgia is common in clients with viral hepatitis. Other symptoms of viral hepatits include lethargy, flulike symptoms, anorexia, N/V, abdominal pain, diarrhea, constipation, and fever. The others are not symptoms of viral hepatitis."

"A client with acute hepatitis is prescribed lactulose. The nurse knows this medication will: A. Prevent the absorption of ammonia from the bowel. B. Prevent hypoglycemia. C. Remove bilirubin from the blood. D. Mobilize iron stores from the liver"

"Correct Answer: A Rationale: Lactulose helps prevent the absorption of ammonia from the bowel because it will cause frequent bowel movements, which facilitates the removal of ammonia from the intestines."

The most common cause of esophageal varices includes which of the following? a) Portal hypertension b) Asterixis c) Jaundice d) Ascites

A) Portal hypertension Esophageal varices are almost always caused by portal hypertension, which results from obstruction of the portal circulation within the damaged liver. Jaundice occurs when the bilirubin concentration in the blood is abnormally elevated. Ascites results from circulatory changes within the diseased liver. Asterixis is an involuntary flapping movement of the hands associated with metabolic liver dysfunction.

A client with acute liver failure exhibits confusion, a declining level of consciousness, and slowed respirations. The nurse finds him very difficult to arouse. The diagnostic information which best explains the client's behavior is: a) subnormal serum glucose and elevated serum ammonia levels. b) subnormal clotting factors and platelet count. c) elevated liver enzymes and low serum protein level. d) elevated blood urea nitrogen and creatinine levels and hyperglycemia.

A) Subnormal serum glucose and elevated serum ammonia levels In acute liver failure, serum ammonia levels increase because the liver can't adequately detoxify the ammonia produced in the GI tract. In addition, serum glucose levels decline because the liver isn't capable of releasing stored glucose. Elevated serum ammonia and subnormal serum glucose levels depress the level of a client's consciousness. Elevated liver enzymes, low serum protein level, subnormal clotting factors and platelet count, elevated blood urea nitrogen and creatine levels, and hyperglycemia aren't as directly related to the client's level of consciousness.

A patient has been admitted to the step-down unit with a diagnosis of liver malignancy. The critical care nurse caring for this patient is aware that an early manifestation of liver malignancy, in addition to pain, is what? A) A continuous dull ache in the right upper quadrant, epigastrium, or back B) Weight gain C) An increase in appetite D) Jaundice

ANS: A The early manifestations of malignancy of the liver include pain and a continuous dull ache in the right upper quadrant epigastrium or back. Weight loss, anorexia, and anemia may occur. Jaundice is present only if the larger bile ducts are occluded by the pressure of malignant nodules in the hilum of the liver.

The nursing instructor is talking about hepatitis with her clinical group. What would the instructor teach the students is the best method to prevent the transmission of the hepatitis E virus? A) Following proper hand-washing techniques B) Avoiding chemicals that are toxic to the liver C) Wearing a condom during sexual relations D) Isolating yourself from your family members

ANS: A Avoiding contact with the hepatitis E virus through good hygiene, including hand-washing, is the major method of prevention. Hepatitis E is transmitted by the fecal-oral route, principally through contaminated water in areas with poor sanitation. It is not necessary for the patient to isolate himself or herself from family members. Condom use is advised for patients with blood-borne hepatitis, such as hepatitis B, C, and D. Nonviral hepatitis is caused by the toxic effects of certain chemicals on the body and does not require transmission precautions.

After a liver transplant, evidence-based guidelines are implemented to prevent the development of what? A) VAP B) ESLD C) MELD D) OLT

ANS: A Evidence-based practice guidelines are implemented to prevent the development of VAP in the postoperative liver transplant recipient. ESLD means end-stage liver disease; MELD is model of end-stage liver disease; orthotopic liver transplantation is OLT.

You are caring for a patient with advanced cirrhosis. You know that the most significant source of bleeding in a patient with cirrhosis is what? A) Portal hypertension B) Esophageal varices C) Hemolytic jaundice D) Ascites

ANS: B Once esophageal varices form, they are increase in size and eventually bleed; in cirrhosis, they are the most significant source of bleeding. Portal hypertension, hemolytic jaundice, and ascites are not sources of bleeding.

An 87-year-old client is in the ICU where you practice nursing. He was admitted for critical care due to his esophageal varices and his precarious physical condition. Which of the following could result in causing his varices to hemorrhage? a) All options are correct b) Little protective tissue to protect fragile veins c) Chemical irritation d) Rough food

All options are correct Explanation: Esophageal varices overfill as a result of portal hypertension. They are especially vulnerable to bleeding because they lie superficially in the mucosa, contain little protective elastic tissue, and are easily traumatized by rough food or chemical irritation.

"A patient with hepatitis B is being discharged in 2 days. In the discharge teching plan the nurse should include instructions to: a. Avoid alcohol for the first 3 weeks b. Use a condom during sexual intercourse c. Have family members get an injection of immunoglobin d. Follow a low-protein, moderate-carbohydrate, moderate-fat diet

Answer B Hepatitis B virus may be transmitted by mucosal exposure to infectious blood, blood products, or other body fluids (e.g., semen, vaginal secretions, saliva). Hepatitis B is a sexually transmitted disease that is acquired through unprotected sex with an infected person. Condom use should be taught to patients to prevent transmission of hepatitis B.

"To prevent the spread of hepatitis A virus (HAV) infection the nurse is especially careful when A. Disposing of food trays B. Emptying bed pans C. Taking an oral temperature D. Changing IV

B is the correct answer. HAV is transmitted primarily person-to-person by the fecal-oral route. Food can be a method of transmission but needs to be fecally contaminated. Since the transmission of hepatitis A is fecal-oral and not saliva or blood like hepatitis B, taking a temperature and changing IV tubing would not spread hepatitis A.

Patients with chronic liver dysfunction have problems with insufficient vitamin intake. Which of the following may occur as a result of vitamin C deficiency? a) Beriberi b) Scurvy c) Night blindness d) Hypoprothrombinemia

B) Scurvy Scurvy may result from a vitamin C deficiency. Night blindness, hypoprothrombinemia, and beriberi do not result from a vitamin C deficiency.

While palpating a client's right upper quadrant (RUQ), the nurse would expect to find which structure? a) Spleen b) Appendix c) Liver d) Sigmoid colon

C) Liver The RUQ contains the liver, gallbladder, duodenum, head of the pancreas, hepatic flexure of the colon, portions of the ascending and transverse colon, and a portion of the right kidney. The sigmoid colon is located in the left lower quadrant; the appendix, in the right lower quadrant; and the spleen, in the left upper quadrant.

The family of a patient newly diagnosed with hepatitis A asks the nurse what they can do to prevent becoming ill themselves. Which of the following responses by the nurse is most appropriate? A) "The hepatitis vaccine will provide immunity from this exposure and future exposures." B) "I am afraid there is nothing you can do since the patient was infectious before admission." C) "You will need to be tested first to make sure you don't have the virus before we can treat you." D) "An injection of immunoglobulin will need to be given to prevent or minimize the effects from this exposure."

D) An injection of immunoglobulin will need to be given to prevent or minimize the effects from this exposure Immunoglobulin provides temporary (1-2 months) passive immunity and is effective for preventing hepatitis A if given within 2 weeks after exposure. It may not prevent infection in all persons, but it will at least modify the illness to a subclinical infection. The hepatitis vaccine is only used for preexposure prophylaxis.

Which type of jaundice seen in adults is the result of increased destruction of red blood cells? a) Obstructive b) Nonobstructive c) Hepatocellular d) Hemolytic

D) Hemolytic Hemolytic jaundice results because, although the liver is functioning normally, it cannot excrete the bilirubin as quickly as it is formed. Obstructive jaundice is the result of liver disease. Nonobstructive jaundice occurs with hepatitis. Hepatocellular jaundice is the result of liver disease.

The mode of transmission of hepatitis A virus (HAV) includes which of the following? a) Fecal-oral b) Blood c) Semen d) Saliva

Fecal-oral Explanation: The mode of transmission of hepatitis A virus (HAV) occurs through fecal-oral route, primarily through person to person contact and/or ingestion of fecal contaminated food or water. Hepatitis B virus (HBV) is transmitted primarily through blood. HBV can be found in blood, saliva, semen, and can be transmitted through mucous membranes and breaks in the skin.

A nursing student is reviewing for an upcoming anatomy and physiology examination. Which of the following would the student correctly identify as a function of the liver? Select all that apply. a) Carbohydrate metabolism b) Glucose metabolism c) Zinc storage d) Ammonia conversion e) Protein metabolism

Glucose metabolism • Ammonia conversion • Protein metabolism Explanation: Functions of the liver include the metabolism of glucose, protein, fat, and drugs; conversion of ammonia; storage of vitamins and iron; formation of bile; and excretion of bilirubin. The liver is not responsible for the metabolism of carbohydrates or the storage of zinc. (less)

A physician orders spironolactone (Aldactone), 50 mg by mouth four times daily, for a client with fluid retention caused by cirrhosis. Which finding indicates that the drug is producing a therapeutic effect? a) Serum sodium level of 135 mEq/L b) Blood pH of 7.25 c) Serum potassium level of 3.5 mEq/L d) Loss of 2.2 lb (1 kg) in 24 hours

Loss of 2.2 lb (1 kg) in 24 hours Explanation: Daily weight measurement is the most accurate indicator of fluid status; a loss of 2.2 lb (1 kg) indicates loss of 1 L of fluid. Because spironolactone is a diuretic, weight loss is the best indicator of its effectiveness. This client's serum potassium and sodium levels are normal. A blood pH of 7.25 indicates acidosis, an adverse reaction to spironolactone.

When performing a physical examination on a client with cirrhosis, a nurse notices that the client's abdomen is enlarged. Which of the following interventions should the nurse consider? a) Ask the client about food intake. b) Report the condition to the physician immediately. c) Provide the client with nonprescription laxatives. d) Measure abdominal girth according to a set routine.

Measure abdominal girth according to a set routine. Explanation: If the abdomen appears enlarged, the nurse measures it according to a set routine. The nurse reports any change in mental status or signs of gastrointestinal bleeding immediately. It is not essential for the client to take laxatives unless prescribed. The client's food intake does not affect the size of the abdomen in case of cirrhosis.

The most common cause of esophageal varices includes which of the following? a) Ascites b) Portal hypertension c) Asterixis d) Jaundice

Portal hypertension Explanation: Esophageal varices are almost always caused by portal hypertension, which results from obstruction of the portal circulation within the damaged liver. Jaundice occurs when the bilirubin concentration in the blood is abnormally elevated. Ascites results from circulatory changes within the diseased liver. Asterixis is an involuntary flapping movement of the hands associated with metabolic liver dysfunction.

Which of the following is the most effective strategy to prevent hepatitis B infection? a) Vaccine b) Barrier protection during intercourse c) Avoid sharing toothbrushes d) Covering open sores

Vaccine Explanation: The most effective strategy to prevent hepatitis B infection is through vaccination. Recommendations to prevent transmission of hepatitis B include vaccination of sexual contacts of individuals with chronic hepatitis, use of barrier protection during sexual intercourse, avoidance of sharing toothbrushes, razors with others, and covering open sores or skin lesions.

"A patient contracts hepatitis from contaminated food. During the acute (icteric) phase of the patient's illness, the nurse would expect serologic testing to reveal... a. hepatitis B surface antigen (HBsAg). b. anti-hepatitis B core immunoglobulin M (anti-HBc IgM). c. anti-hepatitis A virus immunoglobulin G (anti-HAV IgG). d. anti-hepatitis A virus immunoglobulin M (anti-HAV IgM)."

"ANSWER: D Rationale: Hepatitis A is transmitted through the oral-fecal route, and antibody to HAV IgM appears during the acute phase of hepatitis A. The patient would not have antigen or antibodies for hepatitis B. Anti-HAV IgG would indicate past infection and lifelong immunity."

Postnectrotic cirrhosis is a late result of what? A) Acute bacterial hepatitis B) Hemolytic jaundice C) Acute viral hepatitis D) Chronic biliary obstruction

ANS: C Postnecrotic cirrhosis, in which there are broad bands of scar tissue, is a late result of a previous bout of acute viral hepatitis. Therefore options A, B, and D are incorrect.

"Several children at a daycare center have been infected with hepatitis A virus. Which instruction by the nurse would reduce the risk of hepatitis A to the other children and staff members? "1. Hand washing after diaper changes 2. Isolation of the sick children 3. Use of masks during contact with the children 4. Sterilization of all eating utensils"

Answer 1: Rationale: children in day care centers are at risk for hepatits A infection which is transmitted via fecal-oral route due to poor hand hygeine practices and poor sanitation. Isolation of sick children, use of mask during contact, and sterilization of all eating utensils would not be useful in breaking the chain of infection.

"The female nurse sticks herself with a dirty needle. Which action should the nurse implement first? 1.Notify the infection control nurse. 2.Cleanse the area with soap and water. 3.Request post-exposure prophylaxis. 4.Check the hepatitis status of the client.

Answer 2. The nurse should first clean the needle stick with soap and water to help remove any virus that is on the skin

A nurse is caring for a client newly diagnosed with hepatitis A. Which statement by the client indicates the need for further teaching? a) "I'll wash my hands often." b) "How did this happen? I've been faithful my entire marriage." c) "I'll take all my medications as ordered." d) "I'll be very careful when preparing food for my family."

B) How did this happen? I've been faithful my entire marriage The client requires further teaching if he suggests that he acquired the virus through sexual contact. Hepatitis A is transmitted by the oral-fecal route or through ingested food or liquid that's contaminated with the virus. Hepatitis A is rarely transmitted through sexual contact. Clients with hepatitis A need to take every effort to avoid spreading the virus to other members of their family with precautions such as preparing food carefully, washing hands often, and taking medications as ordered.

The nurse is preparing to interview a client with cirrhosis. Based on an understanding of this disorder, which question would be most important to include? a) "What type of over-the-counter pain reliever do you use?" b) "How often do you drink alcohol?" c) "Have you had an infection recently?" d) "Does your work expose you to chemicals?"

B) How often do you drink alcohol? The most common type of cirrhosis results from chronic alcohol intake and is frequently associated with poor nutrition. Although it can follow chronic poisoning with chemicals or ingestion of hepatotoxic drugs such as acetaminophen, asking about alcohol intake would be most important. Asking about an infection or exposure to hepatotoxins or industrial chemicals would be important if the client had postnecrotic cirrhosis.

Which of the following medications would the nurse expect the physician to order for a client with cirrhosis who develops portal hypertension? a) Kanamycin (Kantrex) b) Spironolactone (Aldactone) c) Cyclosporine (Sandimmune) d) Lactulose (Cephulac)

B) Spironlactone (Aldactone) For portal hypertension, a diuretic usually an aldosterone antagonist such as spironolactone (Aldactone) is ordered. Kanamycin (Kantrex) would be used to treat hepatic encephalopathy to destroy intestinal microorganisms and decrease ammonia production. Lactulose would be used to reduce serum ammonia concentration in a client with hepatic encephalopathy. Cyclosporine (Sandimmune) would be used to prevent graft rejection after a transplant.

"The client is admitted to the hospital with viral hepatitis, complaining of ""no appetite"" and ""losing my taste for food."" What instruction should the nurse give the client to provide adequate nutrition? "1. Select foods high in fat 2. Increase intake of fluids, including juices 3. Eat a good supper when anorexia is not as severe 4. Eat less often, preferbly only three large meals daily"

Correct 2: Rationale: Although no specific diet is required to treat viral hepatitis, it is recommended tht clients consume a low-fat diet because fat may be poorly tolerated because of decreased bile production. Small frequent meals are preferable and may even prevent nausea. Frequently, appetite is better in the morning so it is easier to eat a good breakfast. An adequate fluid intake of 2500 to 3000 mL/day that includes nutritional juices is also important.

A college student is required to be inculated for hepatitis before starting college. The nurse recognizes that he will be inoculated for: 1. Hepatitis A 2. Hepatitis B 3. Hepatitis C 4. Hepatitis D

Correct Answer 2 Hepatitis B is considered a significant sexually transmitted disease and is seen in all age groups. A vaccine is available for this type of hepatitis.

When planning care for a patient with cirrhosis, the nurse will give highest priority to which of the following nursing diagnoses? A) Imbalanced nutrition: less than body requirements B) Impaired skin integrity related to edema, ascites, and pruritus C) Excess fluid volume related to portal hypertension and hyperaldosteronism D) Ineffective breathing pattern related to pressure on diaphragm and reduced lung volume

D) Ineffective breathing pattern related to pressure on diaphragm and reduced lung volume Although all of these nursing diagnoses are appropriate and important in the care of a patient with cirrhosis, airway and breathing are always the highest priorities.

Which of the following the are early manifestations of liver cancer? Select all that apply. a) Fever b) Continuous aching in the back c) Vomiting d) Pain e) Increased appetite f) Jaundice

• Pain • Continuous aching in the back Explanation: Early manifestations of liver cancer include pain and continuous dull aching in the right upper quadrant epigastrium or back. Weight loss, anorexia, and anemia may occur. Jaundice is present only if the larger bile ducts are occluded by the pressure of malignant nodules in the hilum of the liver. Fever and vomiting are not associated manifestations.

The family of a patient newly diagnosed with hepatitis A asks the nurse what they can do to prevent becoming ill. Which of the following responses by the nurse is most appropriate? "A) The hepatitis vaccine will provide immunity from this exposure and future exposures."" B) I am afraid there is nothing you can do since the patient was infectious before admission."" C) You will need to be tested first to make sure you don't have the virus before we can treat you."" D) An injection of immunoglobulin will need to be given to prevent or minimize the effects of this exposure."""

"Correct Answer: D Rationale: Immunoglobulin provides temporary (1-2 months) passive immunity and is effective for preventing hepatitis A if given within 2 weeks of exposure. It may not prevent an infection in all persons, but it will at least modify the illness to a subclinical infection. The hepatitis vaccine is only used for preexposure prophylaxis."

A client with liver and renal failure has severe ascites. On initial shift rounds, his primary nurse finds his indwelling urinary catheter collection bag too full to store more urine. The nurse empties more than 2,000 ml from the collection bag. One hour later, she finds the collection bag full again. The nurse notifies the physician, who suspects that a bladder rupture is allowing the drainage of peritoneal fluid. The physician orders a urinalysis to be obtained immediately. The presence of which substance is considered abnormal? a) Albumin b) Creatinine c) Urobilinogen d) Chloride

Albumin Explanation: Albumin is an abnormal finding in a routine urine specimen. Ascites present in liver failure contain albumin; therefore, if the bladder ruptured, ascites containing albumin would drain from the indwelling urinary catheter because the catheter is no longer contained in the bladder. Creatinine, urobilinogen, and chloride are normally found in urine.

A group of nurses have attended an inservice on the prevention of occupationally acquired diseases that affect healthcare providers. What action has the greatest potential to reduce a nurse's risk of acquiring hepatitis C in the workplace? A) Disposing of sharps appropriately and not recapping needles B) Performing meticulous hand hygiene at the appropriate moments in care C) Adhering to the recommended schedule of immunizations D) Wearing an N95 mask when providing care for patients on airborne precautions

Ans: A Feedback: HCV is bloodborne. Consequently, prevention of needlestick injuries is paramount. Hand hygiene, immunizations and appropriate use of masks are important aspects of overall infection control, but these actions do not directly mitigate the risk of HCV.

A patient with a liver mass is undergoing a percutaneous liver biopsy. What action should the nurse perform when assisting with this procedure? A) Position the patient on the right side with a pillow under the costal margin after the procedure. B) Administer 1 unit of albumin 90 minutes before the procedure as ordered. C) Administer at least 1 unit of packed red blood cells as ordered the day before the scheduled procedure. D) Confirm that the patient's electrolyte levels have been assessed prior to the procedure.

Ans: A Feedback: Immediately after a percutaneous liver biopsy, assist the patient to turn onto the right side and place a pillow under the costal margin. Prior administration of albumin or PRBCs is unnecessary. Coagulation tests should be performed, but electrolyte analysis is not necessary.

A patient has developed hepatic encephalopathy secondary to cirrhosis and is receiving care on the medical unit. The patient's current medication regimen includes lactulose (Cephulac) four times daily. What desired outcome should the nurse relate to this pharmacologic intervention? A) Two to 3 soft bowel movements daily B) Significant increase in appetite and food intake C) Absence of nausea and vomiting D) Absence of blood or mucus in stool

Ans: A Feedback: Lactulose (Cephulac) is administered to reduce serum ammonia levels. Two or three soft stools per day are desirable; this indicates that lactulose is performing as intended. Lactulose does not address the patient's appetite, symptoms of nausea and vomiting, or the development of blood and mucus in the stool.

A triage nurse in the emergency department is assessing a patient who presented with complaints of general malaise. Assessment reveals the presence of jaundice and increased abdominal girth. What assessment question best addresses the possible etiology of this patient's presentation? A) "How many alcoholic drinks do you typically consume in a week?" B) "To the best of your knowledge, are your immunizations up to date?" C) "Have you ever worked in an occupation where you might have been exposed to toxins?" D) "Has anyone in your family ever experienced symptoms similar to yours?"

Ans: A Feedback: Signs or symptoms of hepatic dysfunction indicate a need to assess for alcohol use. Immunization status, occupational risks, and family history are also relevant considerations, but alcohol use is a more common etiologic factor in liver disease.

A nurse is caring for a patient with cirrhosis secondary to heavy alcohol use. The nurse's most recent assessment reveals subtle changes in the patient's cognition and behavior. What is the nurse's most appropriate response? A) Ensure that the patient's sodium intake does not exceed recommended levels. B) Report this finding to the primary care provider due to the possibility of hepatic encephalopathy. C) Inform the primary care provider that the patient should be assessed for alcoholic hepatitis. D) Implement interventions aimed at ensuring a calm and therapeutic care environment.

Ans: B Feedback: Monitoring is an essential nursing function to identify early deterioration in mental status. The nurse monitors the patient's mental status closely and reports changes so that treatment of encephalopathy can be initiated promptly. This change in status is likely unrelated to sodium intake and would not signal the onset of hepatitis. A supportive care environment is beneficial, but does not address the patient's physiologic deterioration.

A patient has been diagnosed with advanced stage breast cancer and will soon begin aggressive treatment. What assessment findings would most strongly suggest that the patient may have developed liver metastases? A) Persistent fever and cognitive changes B) Abdominal pain and hepatomegaly C) Peripheral edema unresponsive to diuresis D) Spontaneous bleeding and jaundice

Ans: B Feedback: The early manifestations of malignancy of the liver include pain—a continuous dull ache in the right upper quadrant, epigastrium, or back. Weight loss, loss of strength, anorexia, and anemia may also occur. The liver may be enlarged and irregular on palpation. Jaundice is present only if the larger bile ducts are occluded by the pressure of malignant nodules in the hilum of the liver. Fever, cognitive changes, peripheral edema, and bleeding are atypical signs.

A nurse has entered the room of a patient with cirrhosis and found the patient on the floor. The patient states that she fell when transferring to the commode. The patient's vital signs are within reference ranges and the nurse observes no apparent injuries. What is the nurse's most appropriate action? A) Remove the patient's commode and supply a bedpan. B) Complete an incident report and submit it to the unit supervisor. C) Have the patient assessed by the physician due to the risk of internal bleeding. D) Perform a focused abdominal assessment in order to rule out injury.

Ans: C Feedback: A fall would necessitate thorough medical assessment due to the patient's risk of bleeding. The nurse's abdominal assessment is an appropriate action, but is not wholly sufficient to rule out internal injury. Medical assessment is a priority over removing the commode or filling out an incident report, even though these actions are appropriate.

A patient with portal hypertension has been admitted to the medical floor. The nurse should prioritize which of the following assessments related to the manifestations of this health problem? A) Assessment of blood pressure and assessment for headaches and visual changes B) Assessments for signs and symptoms of venous thromboembolism C) Daily weights and abdominal girth measurement D) Blood glucose monitoring q4h

Ans: C Feedback: Obstruction to blood flow through the damaged liver results in increased blood pressure (portal hypertension) throughout the portal venous system. This can result in varices and ascites in the abdominal cavity. Assessments related to ascites are daily weights and abdominal girths. Portal hypertension is not synonymous with cardiovascular hypertension and does not create a risk for unstable blood glucose or VTE.

A nurse is caring for a patient with hepatic encephalopathy. The nurse's assessment reveals that the patient exhibits episodes of confusion, is difficult to arouse from sleep and has rigid extremities. Based on these clinical findings, the nurse should document what stage of hepatic encephalopathy? A) Stage 1 B) Stage 2 C) Stage 3 D) Stage 4

Ans: C Feedback: Patients in the third stage of hepatic encephalopathy exhibit the following symptoms: stuporous, difficult to arouse, sleeps most of the time, exhibits marked confusion, incoherent in speech, asterixis, increased deep tendon reflexes, rigidity of extremities, marked EEG abnormalities. Patients in stages 1 and 2 exhibit clinical symptoms that are not as advanced as found in stage 3, and patients in stage 4 are comatose. In stage 4, there is an absence of asterixis, absence of deep tendon reflexes, flaccidity of extremities, and EEG abnormalities.

22. A patient with liver disease has developed jaundice; the nurse is collaborating with the patient to develop a nutritional plan. The nurse should prioritize which of the following in the patient's plan? A) Increased potassium intake B) Fluid restriction to 2 L per day C) Reduction in sodium intake D) High-protein, low-fat diet

Ans: C Feedback: Patients with ascites require a sharp reduction in sodium intake. Potassium intake should not be correspondingly increased. There is no need for fluid restriction or increased protein intake.

Diagnostic testing has revealed that a patient's hepatocellular carcinoma (HCC) is limited to one lobe. The nurse should anticipate that this patient's plan of care will focus on what intervention? A) Cryosurgery B) Liver transplantation C) Lobectomy D) Laser hyperthermia

Ans: C Feedback: Surgical resection is the treatment of choice when HCC is confined to one lobe of the liver and the function of the remaining liver is considered adequate for postoperative recovery. Removal of a lobe of the liver (lobectomy) is the most common surgical procedure for excising a liver tumor. While cryosurgery and liver transplantation are other surgical options for management of liver cancer, these procedures are not performed at the same frequency as a lobectomy. Laser hyperthermia is a nonsurgical treatment for liver cancer.

A nurse is performing an admission assessment of a patient with a diagnosis of cirrhosis. What technique should the nurse use to palpate the patient's liver? A) Place hand under the right lower abdominal quadrant and press down lightly with the other hand. B) Place the left hand over the abdomen and behind the left side at the 11th rib. C) Place hand under right lower rib cage and press down lightly with the other hand. D) Hold hand 90 degrees to right side of the abdomen and push down firmly.

Ans: C Feedback: To palpate the liver, the examiner places one hand under the right lower rib cage and presses downward with light pressure with the other hand. The liver is not on the left side or in the right lower abdominal quadrant.

A nurse is caring for a patient with liver failure and is performing an assessment in the knowledge of the patient's increased risk of bleeding. The nurse recognizes that this risk is related to the patient's inability to synthesize prothrombin in the liver. What factor most likely contributes to this loss of function? A) Alterations in glucose metabolism B) Retention of bile salts C) Inadequate production of albumin by hepatocytes D) Inability of the liver to use vitamin K

Ans: D Feedback: Decreased production of several clotting factors may be partially due to deficient absorption of vitamin K from the GI tract. This probably is caused by the inability of liver cells to use vitamin K to make prothrombin. This bleeding risk is unrelated to the roles of glucose, bile salts, or albumin.

A patient with liver cancer is being discharged home with a biliary drainage system in place. The nurse should teach the patient's family how to safely perform which of the following actions? A) Aspirating bile from the catheter using a syringe B) Removing the catheter when output is 15 mL in 24 hours C) Instilling antibiotics into the catheter D) Assessing the patency of the drainage catheter

Ans: D Feedback: Families should be taught to provide basic catheter care, including assessment of patency. Antibiotics are not instilled into the catheter and aspiration using a syringe is contraindicated. The family would not independently remove the catheter; this would be done by a member of the care team when deemed necessary.

A patient with liver cancer is being discharged home with a hepatic artery catheter in place. The nurse should be aware that this catheter will facilitate which of the following? A) Continuous monitoring for portal hypertension B) Administration of immunosuppressive drugs during the first weeks after transplantation C) Real-time monitoring of vascular changes in the hepatic system D) Delivery of a continuous chemotherapeutic dose

Ans: D Feedback: In most cases, the hepatic artery catheter has been inserted surgically and has a prefilled infusion pump that delivers a continuous chemotherapeutic dose until completed. The hepatic artery catheter does not monitor portal hypertension, deliver immunosuppressive drugs, or monitor vascular changes in the hepatic system.

A nurse is participating in the emergency care of a patient who has just developed variceal bleeding. What intervention should the nurse anticipate? A) Infusion of intravenous heparin B) IV administration of albumin C) STAT administration of vitamin K by the intramuscular route D) IV administration of octreotide (Sandostatin)

Ans: D Feedback: Octreotide (Sandostatin)—a synthetic analog of the hormone somatostatin—is effective in decreasing bleeding from esophageal varices, and lacks the vasoconstrictive effects of vasopressin. Because of this safety and efficacy profile, octreotide is considered the preferred treatment regimen for immediate control of variceal bleeding. Vitamin K and albumin are not administered and heparin would exacerbate, not alleviate, bleeding.

A nurse is amending a patient's plan of care in light of the fact that the patient has recently developed ascites. What should the nurse include in this patient's care plan? A) Mobilization with assistance at least 4 times daily B) Administration of beta-adrenergic blockers as ordered C) Vitamin B12 injections as ordered D) Administration of diuretics as ordered

Ans: D Feedback: Use of diuretics along with sodium restriction is successful in 90% of patients with ascites. Beta-blockers are not used to treat ascites and bed rest is often more beneficial than increased mobility. Vitamin B12 injections are not necessary.

"Which statement by the client diagnosed with hepatitis warrants immediate intervention by the clinic nurse? "1) ""I will not drink any type of beer or mixed drink."" 2)""I will get adequate rest so I don't get exhausted."" 3) ""I had a big hearty breakfast this morning."" 4) ""I took some cough syrup for this nasty head cold.""

Answer 4: "Rationale: 1) The client should avoid all alcohol to prevent further liver damage and promote healing. 2) Rest is needed for healing of the liver and to promote optimum immune function. 3) Clients with hepatitis need increased caloric intake so this is a good statement. 4)The client needs to understand some types of cough syrup have alcohol and all alcohol must be avoided to prevent further injury to the liver; therefore, this statement requires intervention"

A female client who has just been diagnosed with hepatitis A asks, "How could I have gotten this disease?" What is the nurse's best response? "A. "You may have eaten contaminated restaurant food." b. "You could have gotten it by using I.V. drugs." c. "You must have received an infected blood transfusion." d. "You probably got it by engaging in unprotected sex.""

Answer A. Hepatitis A virus typically is transmitted by the oral-fecal route — commonly by consuming food contaminated by infected food handlers. The virus isn't transmitted by the I.V. route, blood transfusions, or unprotected sex. Hepatitis B can be transmitted by I.V. drug use or blood transfusion. Hepatitis C can be transmitted by unprotected sex.

During evaluation of a patient at an outpatient clinic, the nurse determines that administration of hepatitis B vaccine has been effective when a specimen of the patient's blood reveals: a. HBsAg. b. anti-HBs c. anti-HBc IgM. d. anti-HBc IgG"

Answer B: The presence of surface antibody to HBV (anti-HBs) is a marker of a positive response to the vaccine. The other laboratory values indicate current infection with HBV

"During an admission assessment, the nurse notes a client with hepatitis exhibits all of the following signs or symptoms. Which one is not related to hepatitis? "A. Anorexia B. Bloody stools C. Dark urine D. Yellow sclera"

Answer: B "RATIONALE (A) Anorexia is an expected assessment finding with hepatitis. (B) Rectal bleeding is not related to hepatitis. Further assessment 358 Clinical Specialties: Content Reviews and Testsis needed to identify the cause. (C) Dark urine is an expected assessment finding with hepatitis and is a result of increased serum bilirubin being excreted by the kidneys. (D) Yellow sclera is a sign of jaundice and is an expected assessment finding with hepatitis. Jaundice is caused by increased serum bilirubin"

A client is suspected of having hepatitis. Which diagnostic test result will assist in confirming this diagnosis? a. Elevated hemoglobin level b. Elevated serum bilirubin level c. Elevated blood urea nitrogen level d. Decreased erythrocycle sedimentation rate"

B. Laboratory indicators of hepatitis include elevated liver enzyme levels, elevated serum bilirubin levels, elevated erythrocyte sedimentation rates, and leukopenia. An elevated blood urea nitrogen level may indicate renal dysfunction. A hemoglobin level is unrelated to this diagnosis.

A 40-year-old woman has been diagnosed with hepatitis A and asks the nurse if other members of her family are at risk for ""catching"" the disease. The nurse's response should be based on the understanding that hepatitis A is transmitted primarily:" "1. during sexual intercourse 2. by contact with infected body secretions. 3. through fecal contamination of food or water. 4. through kissing that involves contact with mucous membranes."

Correct 3: "Hepatitis A is primarily transmitted through ingestion of organisms on fecally contaminated hands, food, or water. Care should be taken in the handling of food and water as well as contaminated items such as bed linens, bedpans, and toilets. Hand hygiene and personal protective equipment such as gloves are important to prevent the spread of infection for hospital personnel. In the home, hand hygiene and good personal hygiene are important to decrease transmission.

The nurse instructs a client diagnosed with hepatitis A about untoward signs and symptoms related to hepatitis that may develop. The one that should be reported to the practitioner is: 1)Fatigue 2)Anorexia 3)Yellow urine 4)Clay-covered stools

Correct 4 1)It is unnecessary to call the practitioner because this symptom is characteristic of hepatitis from the onset of clinical manifestations. 2)It is unnecessary to call the practitioner because this symptom is characteristic of hepatitis from the onset of clinical manifestations. 3) This is the expected color of urine. 4) Clay-colored stools are indicative of hepatic obstruction because bile is prevented from entering the intestines.

A client is admitted with ongoing symptoms of the flu. There are no other obvious signs of illness. This client should be tested for hepatitis because: "a) She could have anicteric hepatitis, which means no jaundice. b) She has a blood pressure of 90/50. c) She was living with a roommate who had similar symptoms. d) She has an allergy to shellfish."

Correct A: (Correct Answer=A) Only about 25 percent of people with acute hepatitis develop jaundice. Patients with anicteric hepatitis may have severely compromised liver function that is overlooked due to lack of jaundice.

"Question: A 40-year-old woman has been diagnosed with hepatitis A and asks the nurse if other members of her family are at risk for ""catching"" the disease. The nurse's response should be based on the understanding that hepatitis A is transmitted primarily 1. During sexual intercourse; 2. By contact with infected body secretions; 3. Through fecal contamination of food or water 4. Through kissing that involves contact with mucous membranes.

Correct Answer 3 Anwer: (3). Rationale: Hepatitis A is primarily transmitted through ingestion of organisms on fecally contaminated hands, food, or water. Care should be taken in the handling of food and water as well as contaminated items such as bed linens, bedpans, and toilets. Hand hygiene and personal protective equipment such as gloves are important to prevent the spread of infection for hospital personnel. In the home, hand hygiene and good personal hygiene are important to decrease transmission. Sexual intercourse (1), contact with infected body secretions (2), and contact through mucous membranes (4) all present higher risk for hepatitis B and C.

What type of precaution should the nurse implement to protect from being exposed to any of the hepatitis viruses? A. Airborne Precautions. B. Standard Precautions. C. Droplet Precautions. D. Exposure Precautions.

Correct Answer B: Standard precautions apply to blood, all body fluids, secretions, and excretions, except sweat, regardless of whether they contain visible blood. Airborne Precautions are only for airborne droplet nuclei or dust particles, Droplet precaution involves large particle droplets in the mucus membranes, and Exposure precaution is not a designated isolation category.

"A patient contracts hepatitis from contaminated food. During the acute (icteric) phase of the patient's illness, the nurse would expect serologic testing to reveal" a. hepatitis B surface antigen (HBsAg). b. anti-hepatitis B core immunoglobulin M (anti-HBc IgM). c. anti-hepatitis A virus immunoglobulin G (anti-HAV IgG). D. anti-hepatitis A virus immunoglobulin M (anti-HAV IgM)."

Correct Answer D "Rationale: Hepatitis A is transmitted through the oral-fecal route, and antibody to HAV IgM appears during the acute phase of hepatitis A. The patient would not have antigen or antibodies for hepatitis B. Anti-HAV IgG would indicate past infection and lifelong immunity."

A sexually active 20-year-old client has developed viral hepatitis. Which of the following statements, if made by the client, would indicate a need for futher teaching? 1. "A condom should be used for sexual intercourse." 2. "I can never drink alcohol again." 3. "I won't go back to work right away." 4. "My close friends should get the vaccine."

Correct Answer: 2. "I can never drink alcohol again." Rationale: To prevent transmission of hepatitis, a condom is advised during sexual intercourse and vaccination of the partner. Alcohol should be avoided because it is detoxified in the liver and may interfere with recover. Rest is especially important until laboratory studies show that liver function has returned to normal. The client's activity is increased gradually

A patient withhepatitis B is being discharged in 2 days. In the discharge teaching plan the nurse should include instructions to "a) Avoid alcohol for the first 3 weeks b) use a condom during sexual intercourse c) have family members get an injection of immunoglobulin d) follow a low-protein, moderate-carbohydrate, moderate fat diet"

Correct B "Correct Answer: B Rationale: Hepatitis B virus may be transmitted by mucosal exposure to infectious blood, blood products, or other body fluids (e.g., semen, vaginal secretions, saliva). Hepatitis B is a sexually transmitted disease that is acquired through unprotected sex with an infected person. Condom use should be taught to patients to prevent transmission of hepatitis B."

"A client with acute hepatitis is prescribed lactulose. The nurse knows this medication will: "a. Mobilize iron stores from the liver. b. Prevent hypoglycemia c. Remove bilirubin from the blood d. Prevent the absorption of ammonia from the bowel.

Correct D Lactulose helps prevent the absorption of ammonia from the bowel because it will cause frequent bowel movements, which facilitates the removal of ammonia from the intestines.

"1. A health care provider who has not been immunized for hepatitis B is exposed to the hepatitis B virus (HBV) through a needle stick from an infected patient. The infection control nurse informs the individual that treatment for the exposure should include: a. baseline hepatitis B antibody testing now and in 2 months. b. active immunization with hepatitis B vaccine. c. hepatitis B immune globulin (HBIG) injection. d. both the hepatitis B vaccine and HBIG injection.

Correct D The recommended treatment for exposure to hepatitis B in unvaccinated individuals is to receive both HBIG and the hepatitis B vaccine, which would provide temporary passive immunity and promote active immunity. Antibody testing may also be done, but this would not provide protection from the exposure.

"Which type of hepatitis is transmitted by the fecal-oral route via contaminated food, water, or direct contact with an infected person? 1. Hepatitis A 2. Hepatitis B 3. Hepatitis C 4. Hepatitis D"

Correct answer: 1 Rationale: 1. The hepatitis A virus is in the stool of infected people for up to 2 weeks before symptoms develop 2. Hepatitis B is spread through contact with infected blood and body fluids 3. Hepatitis C is transmitted through contact with infected blood and body fluids 4. Hepatitis D infection only causes infection in people who are also infected with Hepatitis B or C"

"A client is admitted with ongoing symptoms of the flu. There are no other obvious signs of illness. This client should be tested for hepatitis because: "a) She has an allergy to shellfish. b) She could have anicteric hepatitis, which means no jaundice. c) She was living with a roommate who had similar symptoms. d)She has a blood pressure of 90/50.

Correct answer: B" Only about 25 percent of people with acute hepatitis develop jaundice. Patients with anicteric hepatitis may have severely compromised liver function that is overlooked due to lack of jaundice.

"The client with hepatitis asks the nurse ""I went to an herbalist, who recommended I take milk thistle. What do you think about the herb?"" Which statement is the nurse's best response? "1. ""You are concerned about taking an herb"" 2. ""The herb has been used to treat liver disease"" 3. ""I would not take anything that is not prescribed"" 4. ""Why would you want to take any herbs?""

Correct: 2 "1. This is a therapeutic response and the nurse should provide factual information 2. Milk thistle has an active ingredient, silymarin, which has been used to treat liver disease for more than 2,000 yrs. It is a powerful oxidant and promotes liver cell growth. 3. The nurse should not discourage complementary therapies. 4. This is a judgmental statement, and the nurse should encourage the client to ask questions."

"A patient with hepatitis A is in the acute phase. The nurse plans care for the patient based on the knowledge that: "A. pruritus is a common problem with jaundice in this phase. B. the patient is most likley to transmit the disease during this phase. C. gastrointestinal symptoms are not as severe in hepatitis A as they are in hepatitis B. D. extrahepatic manifestations of glomerulonephritis and polyarteritis are common in this phase."

Correct: A The acute phase of jaundice may be icteric (i.e., symptomatic, including jaundice) or anicteric. Jaundice results when bilirubin diffuses into the tissues. Pruritus sometimes accompanies jaundice. Pruritus is the result of an accumulation of bile salts beneath the skin.

A client is suspected of having hepatitis. Which diagnostic test result will assist in confirming this diagnosis? a. Elevated hemoglobin level B.. Elevated serum bilirubin level c. Elevated blood urea nitrogen level d. Decreased erythrocycle sedimentation rate

Correct: B Answer B. Laboratory indicators of hepatitis include elevated liver enzyme levels, elevated serum bilirubin levels, elevated erythrocyte sedimentation rates, and leukopenia. An elevated blood urea nitrogen level may indicate renal dysfunction. A hemoglobin level is unrelated to this diagnosis

"A patient with hepatitis B is being discharged in 2 days. In the discharge teaching plan the nurse should include instructions to... a. avoid alcohol for the first 3 weeks b. use a condom during sexual intercourse c. have family members get an injection of imunoglobulin d. follow a low-protein, moderate-carbohydrate, moderate-fat diet."

Correct: B Rationale: Hepatitis B virus may be transmitted by mucosal exposure to infectious blood, blood products, or other body fluids (e.g., semen, vaginal secretions, saliva). Hepatitis B is a sexually transmitted disease that is acquired through unprotected sex with an infected person. Condom use should be taught to patients to prevent transmission of hepatitis B.

A female client with hepatitis C develops liver failure and GI hemorrhage. The blood products that would most likely bring about hemostasis in the client are: a. whole blood and albumin. b. platelets and packed red blood cells. c. fresh frozen plasma and whole blood. D.cryoprecipitate and fresh frozen plasma.

Correct: D Answer D. The liver is vital in the synthesis of clotting factors, so when it's diseased or dysfunctional, as in hepatitis C, bleeding occurs. Treatment consists of administering blood products that aid clotting. These include fresh frozen plasma containing fibrinogen and cryoprecipitate, which have most of the clotting factors. Although administering whole blood, albumin, and packed cells will contribute to hemostasis, those products aren't specifically used to treat hemostasis. Platelets are helpful, but the best answer is cryoprecipitate and fresh frozen plasma.

When collecting an admission history, the nurse identifies that the client prefers fish and crustaceans over other sources of protein. When planning discharge teaching for this client the nurse should include the fact that the cooked food most likely to remain contaminated by the virus that causes Hep A is A) canned tuna B) broiled shrimp C) baked haddock D) steamed lobster

D) Steamed lobster. The temperature during steaming is never high enough or sustained long enough to kill organisms

Which of the following is an age-related change of the hepatobiliary system? a) Decreased prevalence of gallstones b) Decreased blood flow c) Increased drug clearance capability d) Liver enlargement

Decreased blood flow Explanation: Age-related changes of the hepatobiliary system include decreased blood flow, decreased drug clearance capability, increased presence of gall stones, and a steady decrease in size and weight of the liver.

A nurse is preparing a presentation for a local community group about hepatitis. Which of the following would the nurse include? a) Hepatitis C increases a person's risk for liver cancer. b) Infection with hepatitis G is similar to hepatitis A. c) Hepatitis A is frequently spread by sexual contact. d) Hepatitis B is transmitted primarily by the oral-fecal route.

Hepatitis C increases a person's risk for liver cancer. Explanation: Infection with hepatitis C increases the risk of a person developing hepatic (liver) cancer. Hepatitis A is transmitted primarily by the oral-fecal route; hepatitis B is frequently spread by sexual contact and infected blood. Hepatitis E is similar to hepatitis A whereas hepatitis G is similar to hepatitis C.

The nurse is caring for a client with cirrhosis. Which assessment findings indicate that the client has deficient vitamin K absorption caused by this hepatic disease? a) Dyspnea and fatigue b) Gynecomastia and testicular atrophy c) Purpura and petechiae d) Ascites and orthopnea

Purpura and petechiae Explanation: A hepatic disorder, such as cirrhosis, may disrupt the liver's normal use of vitamin K to produce prothrombin (a clotting factor). Consequently, the nurse should monitor the client for signs of bleeding, including purpura and petechiae. Dyspnea and fatigue suggest anemia. Ascites and orthopnea are unrelated to vitamin K absorption. Gynecomastia and testicular atrophy result from decreased estrogen metabolism by the diseased liver.

Which of the following medications would the nurse expect the physician to order for a client with cirrhosis who develops portal hypertension? a) Kanamycin (Kantrex) b) Cyclosporine (Sandimmune) c) Spironolactone (Aldactone) d) Lactulose (Cephulac)

Spironolactone (Aldactone) Explanation: For portal hypertension, a diuretic usually an aldosterone antagonist such as spironolactone (Aldactone) is ordered. Kanamycin (Kantrex) would be used to treat hepatic encephalopathy to destroy intestinal microorganisms and decrease ammonia production. Lactulose would be used to reduce serum ammonia concentration in a client with hepatic encephalopathy. Cyclosporine (Sandimmune) would be used to prevent graft rejection after a transplant.

The nurse is preparing a care plan for a patient with hepatic cirrhosis. Which of the following nursing diagnoses are appropriate? Select all that apply. a) Activity intolerance related to fatigue, general debility, muscle wasting, and discomfort b) Altered nutrition, more than body requirements, related to decreased activity and bed rest c) Risk for injury related to altered clotting mechanisms d) Disturbed body image related to changes in appearance, sexual dysfunction, and role function e) Urinary incontinence related to general debility and muscle wasting

• Risk for injury related to altered clotting mechanisms • Disturbed body image related to changes in appearance, sexual dysfunction, and role function • Activity intolerance related to fatigue, general debility, muscle wasting, and discomfort Correct Explanation: Risks for injury, activity intolerance, and disturbed body image are priority nursing diagnoses. The appropriate nursing diagnosis related to nutrition would be altered nutrition, less than body requirements, related to chronic gastritis, decreased GI motility, and anorexia. Urinary incontinence is not generally a concern with hepatic cirrhosis.


Related study sets

Chemistry - Enzymes and Cardiac Assessment

View Set

Chapter 16- AMSCO bonus/analysis *page 1*

View Set

Unit 6: Market Structure (1) -- Firms in Competitive Markets

View Set

Chapter 4: Clinical Assessment, Diagnosis, and Treatment

View Set